AIMEE
AIMEE
di susun oleh :
MIFTAH
Problem 1. MATHEMATICS REVOLUTION (MMR)
Let , , and all exceed , and let be a positive number such that
R, , and SURABAYA
. Find.
AIME 1983
Problem 1.
Let , , and all exceed , and let be a positive number such that
, , and . Find .
Solutions
Solution 1
The logarithmic notation doesn't tell us much, so we'll first convert everything to the equivalent
exponential expressions.
, , and .
Therefore, .
Hence,
Problem 2.
Let , where . Determine
the minimum value taken by for in the interval .
Solution
It is best to get rid of the absolute value first.
Under the given circumstances, we notice that , ,
and .
Adding these together, we find that the sum is equal to , of which the minimum value is
attained when .
Edit: can equal or (for example, if and
, ). Thus, our two "cases" are (if )
and (if ). However, both of these cases give us as the minimum value
for , which indeed is the answer posted above.
Also note the lowest value occurs when because this make the first two
requirements . It is easy then to check that 15 is the minimum value.
Problem 3.
What is the product of the real roots of the equation ?
Solutions
Solution 1
If we expand by squaring, we get a quartic polynomial, which isn't always the easiest thing to
deal with.
Solution 2
We begin by noticing that the polynomial on the left is less than the polynomial under the
radical sign. Thus:
Letting , we have .
Because the square root of a real number can't be negative, the only possible is .
Substituting that in, we have
Problem 4.
A machine shop cutting tool is in the shape of a notched circle, as shown. The radius of the
circle is cm, the length of is 6 cm, and that of is 2 cm. The angle is a right
angle. Find the square of the distance (in centimeters) from to the center of the circle.
Solution
Solution 1
Because we are given a right angle, we look for ways to apply the Pythagorean Theorem. Let
the foot of the perpendicular from to be and let the foot of the perpendicular
from to the line be . Let and . We're trying to find .
Solution 2
Drop perpendiculars from to ( ), to ( ), and to ( ). Also, draw the
midpoint of .
Then the problem is trivialized. Why?
First notice that by computation, is a isosceles triangle;
thus . Then, notice that . Thus the two blue
triangles are congruent.
So, . As , we subtract and get .
Then the Pythagorean Theorem shows .
Problem 5.
Suppose that the sum of the squares of two complex numbers and is and the sum of the
cubes is . What is the largest real value that can have?
Solution
Solution 1
One way to solve this problem seems to be by substitution.
and
Because we are only left with and , substitution won't be too bad.
Let and .
We get and
Because we want the largest possible , let's find an expression for in terms of .
.
Substituting, . Factored, (the Rational
Root Theorem may be used here, along with synthetic division)
Now, evaluating the real part of , which equals (ignoring the odd
powers of , since they would not result in something in the form of ):
d
Since we know that , it can be plugged in for in the above equation to yield:
Since the problem is looking for to be a positive integer, only positive half-
integers (and whole-integers) need to be tested. From the Rational Roots
theorem, all fail, but does work. Thus, the real part of both
numbers is , and their sum is
Solution 3
Start by assuming x and y were roots of some polynomial of the form So
Problem 6.
Let equal . Determine the remainder upon dividing by .
Solution
Solution 1
First, we try to find a relationship between the numbers we're provided with and . We realize
that and both and are greater or less than by .
Thus .
Alternatively, we could have noted that . This way, we
have , and can finish the same way.
Problem 7.
Twenty five of King Arthur's knights are seated at their customary round table. Three of them
are chosen - all choices being equally likely - and are sent of to slay a troublesome dragon.
Let be the probability that at least two of the three had been sitting next to each other.
If is written as a fraction in lowest terms, what is the sum of the numerator and the
denominator?
Solution
Solution 1
We can use Complementary counting by finding the probability that none are sitting next to
each other and subtracting it from .
Imagine the other (indistinguishable) people are already seated, and fixed into place.
We will place , , and with and without the restriction.
There are places to place , followed by places to place , and places to
place after and . Hence, there are ways to place in between
these people with restrictions.
Without restrictions, there are places to place , followed by places to place ,
and places to place after and . Hence, there are ways to
place in between these people without restrictions.
There are ways to pick a pair of knights from the trio, and there are ways to
determine which order they are seated. Since these two knights must be attached, we let them
be a single entity, so there are configurations for the entities.
However, this overcounts the instances in which the trio sits together; when all three knights sit
together, then two of the pairs from the previous case are counted. However, we only want to
count this as one case, so we need to subtract the number of instances in which the trio sits
together (as a single entity). There are ways to determine their order, and there
are configurations.
of 24, so the probability of this is . We do not need to consider the third knight.
Case 2: The second knight sits two spaces from the first knight. There are 2 possible places
for this out of 24, so the probability is . Then there are 3 places out of a remaining 23 for the
places for this out of 24, so the probability is . Then there are four places to put the last
Problem 8.
Solution
Solution
Solution 1
This results in .
Thus, if , then the minimum is obviously . We show this possible with the
same methods in Solution 1; thus the answer is .
Solution 3
answer is .
Solution 4 (also uses calculus)
As above, let . Add to the expression and subtract ,
on the given interval, we can ignore the negative result. Plugging into our expression
for , we have .
Problem 10.
The numbers , , and have something in common. Each is a four-digit number
beginning with that has exactly two identical digits. How many such numbers are there?
Solution
Solution 1
Suppose the two identical digits are both one. Since the thousands digits must be one, the
other one can be in only one of three digits,
Because the number must have exactly two identical digits, , , and . Hence,
there are numbers of this form.
Suppose the two identical digits are not one. Therefore, consider the following possibilities,
sequence starts with 1 is . This means we can find all possible sequences with one digit
repeated twice, and then divide by .
If we let the three distinct digits of the sequence be and with repeated twice, we can
make a table with all possible sequences:
Problem 11.
The solid shown has a square base of side length . The upper edge is parallel to the base
and has length . All other edges have length . Given that , what is the volume of
the solid?
Solution
Solution 1
First, we find the height of the figure by drawing a perpendicular from the midpoint
of to . The hypotenuse of the triangle is the median of equilateral triangle , and
one of the legs is . We apply the Pythagorean Theorem to find that the height is equal
to .
Next, we complete the figure into a triangular prism, and find the volume, which
is .
Now, we subtract off the two extra pyramids that we included, whose combined volume
is .
is , where S is the side length of the tetrahedron, the volume of our original solid
is:
Problem 12.
The length of diameter is a two digit integer. Reversing the digits gives the length of a
perpendicular chord . The distance from their intersection point to the center is a
positive rational number. Determine the length of .
Solution
Let and . It follows
.
Because is a positive rational number, the quantity cannot contain
any square roots. Either or must be 11. However, cannot be 11, because
both must be digits. Therefore, must equal eleven and must be a perfect square
(since ). The only pair that satisfies this condition is , so our
answer is .
Problem 13.
For and each of its non-empty subsets, an alternating sum is defined as
follows. Arrange the number in the subset in decreasing order and then, beginning with the
largest, alternately add and subtract successive numbers. For example, the alternating sum
for is and for it is simply . Find the sum of all
such alternating sums for .
Solution 1
Let be a non- empty subset of .
Then the alternating sum of plus the alternating sum of with 7 included is 7. In
mathematical terms, . This is true because when we take an alternating
sum, each term of has the opposite sign of each corresponding term of .
Because there are of these pairs, the sum of all possible subsets of our given set is .
However, we forgot to include the subset that only contains , so our answer is
.
Solution 2
Consider a given subset of that contains 7; then there is a subset which contains all
the elements of except for 7, and only those. Since each element of has one element
fewer preceding it than it does in , their signs are opposite; so the sum of the alternating
sums of and is equal to 7. There are subsets containing 7, so our answer
is .
Solution 3
Denote the desired total of alternating sums of an element set with . We are looking
for . Note that all alternating sums of an element set are also alternating sums of
an element set. However, when we go from an to element set, for each subset
with the new element, we are adding the new element and subtracting one of the alternating
sums of the element set. There are subsets of an element set that includes the
new element, giving us the following relationship.
.
Problem 14.
In the adjoining figure, two circles with radii and are drawn with their centers units apart.
At , one of the points of intersection, a line is drawn in such a way that the
chords and have equal length. Find the square of the length of .
Solution
Solution 1
First, notice that if we reflect over we get . Since we know that is
on circle and is on circle , we can reflect circle over to get another circle (centered
at a new point with radius ) that intersects circle at . The rest is just finding lengths:
Since is the midpoint of segment , is a median of triangle . Because we know
that , , and , we can find the third side of the triangle
using Stewart's Theorem or similar approaches. We get . So now we have a
kite with , , and , and all we need is the
length of the other diagonal . The easiest way it can be found is with the Pythagorean
Theorem. Let be the length of . Then
Draw additional lines as indicated. Note that since triangles and are isosceles,
the altitudes are also bisectors, so let .
Subtracting, .
Solution 3
Let . Angles , , and must add up to . By the Law of
Cosines, . Also,
angles and equal and . So we have
Taking the of both sides and simplifying using the cosine addition identity gives .
Solution 4 (quickest)
Let Extend the line containing the centers of the two circles to meet R and
the other side of the circle the large circle.
The line segment consisting of R and the first intersection of the larger circle has length 10.
The length of the diameter of the larger circle be16.
Through power of a point,
Problem 15.
The adjoining figure shows two intersecting chords in a circle, with on minor arc .
Suppose that the radius of the circle is , that , and that is bisected by .
Suppose further that is the only chord starting at which is bisected by . It follows
that the sine of the minor arc is a rational number. If this fraction is expressed as a
Solution 1
Let be any fixed point on circle and let be a chord of circle .
The locus of midpoints of the chord is a circle , with diameter . Generally, the
circle can intersect the chord at two points, one point, or they may not have a point of
intersection. By the problem condition, however, the circle is tangent to BC at point N.
Let M be the midpoint of the chord . From right triangle
, . Thus, .
Notice that the
distance equals (Where is the radius of
, . It follows
, . Thus,
.
From here, we see that is a dilation of about center with ratio ,
so .
Lastly, we apply the formula:
Let the circle be , and its center be labeled . Since BC=6, we can
calculate (by the Pythagorean Theorem) that the distance from to the line is . So we
can let and . Now assume that is any point on the major arc BC,
and any point on the minor arc BC. We may let , where is the
angle measured from the positive axis to the ray . It will also be convenient to
define as the measure of angle XOB.
Firstly, since B must lie in the minor arc AD, we see that . However,
since the midpoint of AD must lie on BC, and the highest coordinate of is , we see that
Given that this equation has only one real root , we study the following function:
First, by the fact that has real solution, it is good to look at its discriminant: must be non-
negative:
It is obvious that this is non-negative. If it is actually zero, then , and . In
this case, . We found a possible case. So we
calculate .
Note to Solution 2
Note: As an AIME problem, it is already done since we have found one possible case.
However, it takes one more step to complete it if we need to say that this is the unique
possibility, without appealing to the AIME Uniqueness Principle.
Suppose that , which means that there can be two real roots of , one lying in the
interval , but another falling out of it. We also see that the average of the two root
is , which is a quantity greater than 0, so the root outside of must be no
less than 3. Spectating the parabolic curve of , which is a "U shaped" curve hitting the
interval once and another time, it is evident that and .
We can just work on the second one:
The only way to satisfy this equation is when (by working on Cauchy-Schwarz
inequality, or just plotting the line to see that point can't go above this line),
which does not make sense from the description of the problem. It means that the point lies
in the half plane above the line , inclusive, and the half plane below the
line , exclusive. It is obviously impossible, by drawing the lines and see that
the intersection of the two half planes does not share any point with the circle.
AIME 1984
Problem 1.
Find the value of if , , is an arithmetic
progression with common difference 1, and .
Solution
Solution 1
One approach to this problem is to apply the formula for the sum of an arithmetic series in order
to find the value of , then use that to calculate and sum another arithmetic series to get our
answer.
A somewhat quicker method is to do the following: for each , we have
. We can substitute this into our given equation to
get . The left-hand side of
this equation is simply , so our desired value
is .
Solution 2
If is the first term, then can be rewritten as:
which is . So, from the first equation, we know . So, the final
answer is:
.
Solution 3
A better approach to this problem is to notice that from that each
element with an odd subscript is 1 from each element with an even subscript. Thus, we note that
the sum of the odd elements must be . Thus, if we want to find the sum of all of the
Problem 2.
The integer is the smallest positive multiple of such that every digit of is either or .
Compute .
Solution
Any multiple of 15 is a multiple of 5 and a multiple of 3.
Any multiple of 5 ends in 0 or 5; since only contains the digits 0 and 8, the units digit of must
be 0.
The sum of the digits of any multiple of 3 must be divisible by 3. If has digits equal to 8, the
sum of the digits of is . For this number to be divisible by 3, must be divisible by 3. We
also know that since is positive. Thus must have at least three copies of the digit 8.
The smallest number which meets these two requirements is 8880. Thus the answer
is .
Problem 3.
A point is chosen in the interior of such that when lines are drawn
through parallel to the sides of , the resulting smaller triangles , , and in the
figure, have areas , , and , respectively. Find the area of .
Solution 1
By the transversals that go through , all four triangles are similar to each other by
the postulate. Also, note that the length of any one side of the larger triangle is equal to the
sum of the sides of each of the corresponding sides on the smaller triangles. We use the
identity to show that the areas are proportional (the sides are proportional and
the angles are equal) Hence, we can write the lengths of corresponding sides of the triangle
as . Thus, the corresponding side on the large triangle is , and the area of the
triangle is .
Solution 2
Alternatively, since the triangles are similar by , then the ratios between the bases and the
heights of each of the three triangles would all be equal. The areas of each of the triangles are
all perfect squares, so we could assume That means that the base of is 4, the
base of is 6, and the base of is 14. Since the quadrilaterals underneath and are both
parallelograms, and opposite sides of a parallelogram are congruent, the base of the large
triangle is . Therefore, the height of the entire triangle would be twelve, so
Problem 4.
Let be a list of positive integers - not necessarily distinct - in which the number appears.
The arithmetic mean of the numbers in is . However, if is removed, the arithmetic
mean of the numbers is . What's the largest number that can appear in ?
Solution
Suppose has members other than 68, and the sum of these members is . Then we're
Problem 5.
Determine the value of if and .
Solution
Solution 1
find that . Doing the same thing with the second equation yields that
. This means that and
that . If we multiply the two equations together, we get
that , so taking the fourth root of that, .
Solution 2
We can simplify our expressions by changing everything to a common base and by pulling
exponents out of the logarithms. The given equations then
Problem 6.
Three circles, each of radius , are drawn with centers at , , and .
A line passing through is such that the total area of the parts of the three circles to
one side of the line is equal to the total area of the parts of the three circles to the other side of
it. What is the absolute value of the slope of this line?
Solution
The line passes through the center of the second circle; hence it is the circle's diameter and
splits the circle into two equal areas. For the rest of the problem, we do not have to worry
about that circle.
Solution 1
Draw the midpoint of (the centers of the other two circles), and call it . If we draw the
feet of the perpendiculars from to the line (call ), we see
that by HA congruency; hence lies on the line. The coordinates
of are .
And .
Solution 3 (non-rigorous)
Consider the region of the plane between and . The parts of the circles
centered at and in this region have equal area. This is by symmetry- the
lines defining the region are 2 units away from the centers of each circle and therefore cut off
congruent segments. We will draw the line in a way that uses this symmetry and makes
identical cuts on the circles. Since is units below the center of the lower circle, we
will have the line exit the region units above the center of the upper circle, at . We
then find that the slope of the line is and our answer is .
(Note: this solution does not feel rigorous when working through it, but it can be checked
easily. In the above diagram, the point is marked. Rotate the aforementioned region of the
plane about point , and the fact that certain areas are equal is evident.)
Problem 7.
The function f is defined on the set of integers and
satisfies
Find .
Solution 1
Define , where the function is performed times. We find
that
. . So we now need to reduce .
Let’s write out a couple more iterations of this function:
So this function
reiterates with a period of 2 for . It might be tempting at first to assume
that is the answer; however, that is not true since the solution occurs slightly
before that. Start at :
Solution 2
We start by finding values of the function right under since they require iteration of the
function.
Soon we realize the for integers either equal or based on it parity. (If
short on time, a guess of or can be taken now.) If is even if is
odd . has even parity, so .
Problem 8.
The equation has complex roots with argument between and in
the complex plane. Determine the degree measure of .
Solution 1
We shall introduce another factor to make the equation easier to solve. If is a root
of , then . The polynomial has all of
its roots with absolute value and argument of the form for integer (the ninth
degree roots of unity). Now we simply need to find the root within the desired range that
satisfies our original equation .
This reduces to either or . But can't be because
if , then . (When we multiplied by at the
beginning, we introduced some extraneous solutions, and the solution with was one of
them.) This leaves .
Solution 2
The substitution simplifies the equation to . Applying the quadratic
Problem 9.
In tetrahedron , edge has length 3 cm. The area of face is and the
area of face is . These two faces meet each other at a angle. Find
the volume of the tetrahedron in .
Solution
tetrahedron is thus .
Problem 10.
Mary told John her score on the American High School Mathematics Examination (AHSME),
which was over . From this, John was able to determine the number of problems Mary
solved correctly. If Mary's score had been any lower, but still over , John could not have
determined this. What was Mary's score? (Recall that the AHSME consists of multiple
choice problems and that one's score, , is computed by the formula ,
where is the number of correct answers and is the number of wrong answers. (Students
are not penalized for problems left unanswered.)
Solution
Let Mary's score, number correct, and number wrong be respectively. Then
.
Therefore, Mary could not have left at least five blank; otherwise, 1 more correct and 4 more
wrong would produce the same score. Similarly, Mary could not have answered at least four
wrong (clearly Mary answered at least one right to have score above 80, or even 30.)
It follows that and ,
so and . So Mary scored at least 119.
To see that no result other than 23 right/3 wrong produces 119, note
that so . But if , then , which was
the result given; otherwise and , but this implies at least 31 questions, a
contradiction. This makes the minimum score .
Problem 11.
A gardener plants three maple trees, four oaks, and five birch trees in a row. He plants them in
random order, each arrangement being equally likely. Let in lowest terms be
the probability that no two birch trees are next to one another. Find .
Solution
First notice that there is no difference between the maple trees and the oak trees; we have
only two types, birch trees and "non-birch" trees. (If you don't believe this reasoning, think
about it. You could also differentiate the tall oak trees from the short oak trees, and the maple
trees with many branches as opposed to those with few branches. Indeed, you could keep
dividing until you have them each in their own category, but in the end it will not change the
probability of the birch trees being near each other. That is, in the end, you multiply the
numerator by the number of ways to arrange the oak and maple trees and you also multiply
the denominator by the number of ways to arrange the oak and maple trees, making them
cancel out.)
The five birch trees must be placed amongst the seven previous trees. We can think of these
trees as 5 dividers of 8 slots that the birch trees can go in, making different ways
to arrange this.
There are total ways to arrange the twelve trees, so the probability
is .
The answer is .
Problem 12.
A function is defined for all real numbers and
satisfies and for all . If is a root
for , what is the least number of roots must have in the
interval ?
Solution
If , then substituting gives .
Similarly, . In particular,
Since is a root, all multiples of are roots, and anything congruent to are also
roots. To see that these may be the only integer roots, observe that the function
Problem 13.
Find the value of
Solution
Solution 1
We know that so we can repeatedly apply the addition
formula, . Let ,
, , and . We have
,
So
and
,
so
these four angles forms the angle of triangle , which has a cotangent of ,
is .
Solution 4
Problem 14.
What is the largest even integer that cannot be written as the sum of two odd composite
numbers?
Solution 1
Take an even positive integer . is either , , or . Notice that the
numbers , , , ... , and in general for nonnegative are odd composites. We now
have 3 cases:
If and is , can be expressed as for some nonnegative .
Note that and are both odd composites.
If and is , can be expressed as for some nonnegative .
Note that and are both odd composites.
If and is , can be expressed as for some nonnegative .
Note that and are both odd composites.
Solution 2
Let be an integer that cannot be written as the sum of two odd composite numbers.
If , then and must all be prime
(or , which yields which does not work).
Thus and form a prime quintuplet. However, only
one prime quintuplet exists as exactly one of those 5 numbers must be divisible by 5.This
prime quintuplet is and , yielding a maximal answer of 38.
Since , which is prime, the answer is .
Problem 15.
Determine if
Solution 1
Since the polynomials are equal at , we can express the difference of the two
polynomials with a quartic polynomial that has roots at , so
Now we can plug in into the polynomial equation. Most terms drop, and we end up with
so that
with a sum of
/*Lengthy proof that any two cubic polynomials in which are equal at 4 values of are
themselves equivalent: Let the two polynomials be and and let them be equal
at . Thus we
have . Also
the polynomial is cubic, but it equals 0 at 4 values of . Thus it must be
equivalent to the polynomial 0, since if it were nonzero it would necessarily be able to be
factored into some nonzero polynomial which would have a
degree greater than or equal to 4, contradicting the statement that is cubic.
Because and are equivalent and must be equal for all .
Post script for the puzzled: This solution which is seemingly unnecessarily redundant in that
it computes and separately before adding them to obtain the final answer is
appealing because it gives the individual values of and which can be plugged
into the given equations to check.
Solution 2
As in Solution 1, we have
Solution 1
Solution 2
Another way to do this is to realize that most of our numbers will be canceled out in the
multiplication in the end, and to just list out the terms of our product and cancel:
Problem 2.
When a right triangle is rotated about one leg, the volume of the cone produced is .
When the triangle is rotated about the other leg, the volume of the cone produced
is . What is the length (in cm) of the hypotenuse of the triangle?
Solution
Let one leg of the triangle have length and let the other leg have length . When we rotate
around the leg of length , the result is a cone of height and radius , and so of
volume . Likewise, when we rotate around the leg of length we get a cone of
Problem 3.
Find if , , and are positive integers which satisfy , where .
Solution
Expanding out both sides of the given equation we
have . Two complex numbers are equal if and only
if their real parts and imaginary parts are equal,
so and . Since are integers, this
means is a divisor of 107, which is a prime number. Thus either or .
If , so , but is not divisible by 3, a
contradiction. Thus we must have , so and (since we
know is positive). Thus .
Problem 4.
A small square is constructed inside a square of area 1 by dividing each side of the unit
square into equal parts, and then connecting the verticesto the division points closest to the
opposite vertices. Find the value of if the the area of the small square is exactly .
Solution 1
The lines passing through and divide the square into three parts, two right triangles and
a parallelogram. Using the smaller side of the parallelogram, , as the base, where the
height is 1, we find that the area of the parallelogram is . By the Pythagorean Theorem,
the longer base of the parallelogram
height . But the height of the parallelogram is the side of the little
square, so . Solving this quadratic equation gives .
Solution 2
Surrounding the square with area are right triangles with hypotenuse (sides of the
large square). Thus, , where is the area of the of the 4 triangles. We can
thus use proportions to solve this problem.
Also,
Thus, Simple
factorization and guess and check gives us .
Problem 5.
A sequence of integers is chosen so that for each .
What is the sum of the first 2001 terms of this sequence if the sum of the first 1492 terms is
1985, and the sum of the first 1985 terms is 1492?
Solution
The problem gives us a sequence defined by a recursion, so let's calculate a few values to get
a feel for how it acts. We aren't given initial values, so let and .
Then , ,
, , and . Since
the sequence is recursively defined by the first 2 terms, after this point it must continue to
repeat. Thus, in particular for all , and so repeating this times, for
all integers and .
Because of this, the sum of the first 1492 terms can be greatly simplified: is
the largest multiple of 6 less than 1492,
so
, where we can make this last step because and so the entire second term of
our expression is zero.
Similarly, since
,
Finally, .
Then by the givens, and so and so
the answer is
Problem 6.
As shown in the figure, triangle is divided into six smaller triangles by lines drawn from
the vertices through a common interior point. The areas of four of these triangles are as
indicated. Find the area of triangle .
Solution
Let the interior point be , let the points on , and be , and , respectively.
Let be the area of and be the area of . Note
that and share the same altitude from , so the ratio of their areas is the
same as the ratio of their bases. Similarly, and share the same altitude
from , so the ratio of their areas is the same as the ratio of their bases. Moreover, the two
pairs of bases are actually the same, and thus in the same ratio. As a result, we
Solution 2
This problem can be done using mass points. Assign B a weight of 1 and realize that many of
the triangles have the same altitude. After continuously using the formulas that state (The sum
of the two weights) = (The middle weight), and (The weight side) = (Other weight) (The
other side), the problem yields the answer
Problem 7.
Assume that , , , and are positive integers such that , , and .
Determine .
Solution
It follows from the givens that is a perfect fourth power, is a perfect fifth power, is
a perfect square and is a perfect cube. Thus, there exist integers and such that
, , and . So . We can factor the left-hand side of
this equation as a difference of two squares, . 19 is a prime
number and so we must have and .
Then and so , and .
Problem 8.
The sum of the following seven numbers is exactly 19: , ,
, , , , . It is desired to replace each by
an integer approximation , , so that the sum of the 's is also 19 and so that ,
the maximum of the "errors" , the maximum absolute value of the difference, is as
small as possible. For this minimum , what is ?
Solution
If any of the approximations is less than 2 or more than 3, the error associated with that
term will be larger than 1, so the largest error will be larger than 1. However, if all of the are
2 or 3, the largest error will be less than 1. So in the best case, we write 19 as a sum of 7
numbers, each of which is 2 or 3. Then there must be five 3s and two 2s. It is clear that in the
best appoximation, the two 2s will be used to approximate the two smallest of the , so our
approximations are and and the largest
error is , so the answer is .
Problem 9.
In a circle, parallel chords of lengths 2, 3, and 4 determine central angles of , ,
and radians, respectively, where . If , which is a positive rational
number, is expressed as a fraction in lowest terms, what is the sum of its numerator and
denominator?
Solution
All chords of a given length in a given circle subtend the same arc and therefore the same
central angle. Thus, by the given, we can re-arrange our chords into a triangle with the circle
as its circumcircle.
This triangle has semiperimeter so by Heron's formula it
so and .
Now, consider the triangle formed by two radii and the chord of length 2. This isosceles
triangle has vertex angle , so by the Law of Cosines,
Problem 10.
How many of the first 1000 positive integers can be expressed in the form
where is a real number, and denotes the greatest integer less than or equal to ?
Solution
We will be able to reach the same number of integers while ranges from 0 to 1 as we will
when ranges from to for any integer (Quick
proof: ).
Since , the answer
must be exactly 50 times the number of integers we will be able to reach as ranges from 0 to
1, including 1 but excluding 0.
Solution 1
Noting that all of the numbers are even, we can reduce this to any real
: We can partition as , and from the previous case we see that works.
Out of these 6 cases, only 3 fails. So between 1 and 10 we can reach only the
integers ; hence our solution is .
Solution 2
As we change the value of , the value of our expression changes only
need only see what happens at the numbers of the form . This gives us
24 calculations to make; we summarize the results here:
Thus, we hit 12 of the first 20 integers and so we hit of the first .
Solution 3
.
There are terms here (we don't actually have to write all of it out; we can just see where
there will be duplicates and subtract accordingly from ). Starting from every integer , we
can keep adding to achieve one higher value for each of these terms, but after raising the last
term, we will have raised the whole sum by while only achieving of those values. We
can conveniently shift the (since it can be achieved) to the position of the so that there
Solution
An ellipse is defined to be the locus of points such that the sum of the distances
between and the two foci is constant. Let , and be
the point of tangency of the ellipse with the -axis. Then must be the point on the axis such
that the sum is minimal. Finding the optimal location for is a classic problem:
for any path from to and then back to , we can reflect the second leg of this path
(from to ) across the -axis. Then our path connects to the reflection of via
some point on the -axis, and this path will have shortest length exactly when our original path
has shortest length. This occurs exactly when we have a straight-line path.
The sum of the two distances and is therefore equal to the length of the
segment , which by the distance formula is
just .
Finally, let and be the two endpoints of the major axis of the ellipse. Then by
symmetry so (because is on the
ellipse), so the answer is .
Problem 12.
Let , , and be the vertices of a regular tetrahedron each of whose edges measures 1
meter. A bug, starting from vertex , observes the following rule: at each vertex it chooses
one of the three edges meeting at that vertex, each edge being equally likely to be chosen,
and crawls along that edge to the vertex at its opposite end. Let be
the probability that the bug is at vertex when it has crawled exactly 7 meters. Find the value
of .
Solution
Solution 1
Let denote the probability that the bug is at after it has crawled meters. Since the
bug can only be at vertex if it just left a vertex which is not , we
have . We also know , so we can quickly
compute , , , ,
Problem 13.
The numbers in the sequence , , , , are of the form ,
where For each , let be the greatest common divisor of and . Find
the maximum value of as ranges through the positive integers.
Solution 1
If denotes the greatest common divisor of and , then we
have . Now assuming
that divides , it must divide if it is going to divide the
entire expression .
Solution 2
We know that and . Since
we want to find the GCD of and , we can use the Euclidean algorithm:
Now, the question is to find the GCD of and . We subtract 100 times
from . This leaves us with . We want this to equal 0, so solving for gives
us . The last remainder is 0, thus is our GCD.
Solution 3
If Solution 2 is not entirely obvious, our answer is the max possible range of .
Using the Euclidean Algorithm on and yields that they are relatively prime. Thus, the
only way the GCD will not be 1 is if the term share factors with the . Using the
Euclidean
Algorithm,
.
Thus, the max GCD is 401.
Problem 14.
In a tournament each player played exactly one game against each of the other players. In
each game the winner was awarded point, the loser got points, and each of the two players
earned point if the game was a tie. After the completion of the tournament, it was found that
exactly half of the points earned by each player were earned against the ten players with the
least number of points. (In particular, each of the ten lowest scoring players earned half of
her/his points against the other nine of the ten). What was the total number of players in the
tournament?
Solution
Let us suppose for convenience that there were players over all. Among the players
not in the weakest 10 there were games played and thus points earned. By the
givens, this means that these players also earned points against our weakest 10. Now,
the total number of points earned was . However, there was one
have so and
and or . Now, note that the top players
got points in total (by our previous calculation) for an average of , while the
bottom 10 got 90 points total, for an average of 9. Thus we must have , so and
the answer is .
Problem 15.
Three 12 cm 12 cm squares are each cut into two pieces and , as shown in the first
figure below, by joining the midpoints of two adjacent sides. These six pieces are then
attached to a regular hexagon, as shown in the second figure, so as to fold into a polyhedron.
What is the volume(in ) of this polyhedron?
Solution
Note that gluing two of the given polyhedra together along a hexagonal face (rotated from
Solution
Let . Then we have , or, by simplifying,
Problem 2.
Evaluate the
product .
Solution
Simplify by repeated application of the difference of squares.
Problem 3.
If and , what is ?
Solution 1
Since is the reciprocal function of :
Thus,
Using the tangent addition formula:
Solution 2
have .
Similarly, we
have .
Dividing:
have
Problem 4.
Determine if , , , , and satisfy the system of equations below.
Solution
Adding all five equations gives
us so
. Subtracting this from the fourth given equation
gives and subtracting it from the fifth given equation gives , so our answer
is
Problem 5.
What is that largest positive integer for which is divisible by ?
Solution
If , . Using the Euclidean algorithm, we
have
, so must divide 900. The greatest integer for
which divides 900 is 890; we can double-check manually and we find that
indeed .
In a similar manner, we can apply synthetic division. We are looking
Solution
Denote the page number as , with . The sum formula shows
is .
Problem 7.
The increasing sequence consists of all those positive integers which
are powers of 3 or sums of distinct powers of 3. Find the term of this sequence.
Solution
Solution 1
Rewrite all of the terms in base 3. Since the numbers are sums of distinct powers of 3, in base
3 each number is a sequence of 1s and 0s (if there is a 2, then it is no longer the sum of
distinct powers of 3). Therefore, we can recast this into base 2 (binary) in order to determine
the 100th number. is equal to , so in binary form we get . However,
we must change it back to base 10 for the answer, which
is .
Solution 2
Notice that the first term of the sequence is , the second is , the fourth is , and so on. Thus
the term of the sequence is . Now out of terms which are of the form +
, of them include and do not. The smallest term that includes , i.e. , is
greater than the largest term which does not, or . So the th term will be , then ,
then , then , and finally
Solution 3
After the nth power of in the sequence, the number of terms after that power but before the
n+1th power of is equal to the number of terms before the nth power because those terms
after the nth power are just the nth power plus all the distinct combinations of powers
of before it, which is just all the terms before it. Adding the powers of and the terms that
come after them, we see that the th term is after , which is the th term. Also, note
that the kth term after the nth power of is equal to the power plus the kth term in the entire
sequence. Thus, the th term is plus the th term. Using the same logic, the th term
is plus the th term, 9. We now have
Problem 8.
Let be the sum of the base logarithms of all the proper divisors (all divisors of a number
excluding itself) of . What is the integer nearest to ?
Solution
Solution 1
The prime factorization of , so there are divisors, of
which are proper. The sum of multiple logarithms of the same base is equal to the logarithm
of the products of the numbers.
Writing out the first few terms, we see that the answer is equal to
Each power
of appears times; and the same goes for . So the overall power
of and is . However, since the question asks
for proper divisors, we exclude , so each power is actually times. The answer is
thus .
Solution 2
Since the prime factorization of is , the number of factors in is . You
can pair them up into groups of two so each group multiplies to . Note
that . Thus, the sum of the logs of the
divisors is half the number of divisors of (since they are asking only for proper
divisors), and the answer is .
Problem 9.
In , , , and . An interior point is then drawn,
and segments are drawn through parallel to the sides of the triangle. If these three
segments are of an equal length , find .
Solution
Solution 1
Notice that has mass . On the other hand, by similar triangles, . Hence by
Hence,
Solution 2
Let the points at which the segments hit the triangle be called as shown
above. As a result of the lines being parallel, all three smaller triangles and the larger triangle
are similar ( ). The remaining three sections
are parallelograms.
Since is a parallelogram, we find , and similarly .
So . Thus . By the same
logic, .
Since , we have the proportion:
Now, .
Solution 3
Define the points the same as above.
Let , , ,
, and
The key theorem we apply here is that the ratio of the areas of 2 similar triangles is the ratio of
a pair of corresponding sides squared.
Let the length of the segment be and the area of the triangle be , using the theorem, we
get:
, , adding all these
together and using we
get
Using corresponding angles from parallel lines, it is easy to show that ,
since and are parallelograms, it is easy to show that
Now we have the side length ratio, so we have the area
and
Substituting these into our initial equation, we
have
answer follows after some hideous computation.
Solution 4
Refer to the diagram in solution 2; let , , and . Now,
note that , , and are similar, so through some similarities we find
that .
Similarly, we find that and ,
gives
.
Solution 5
Refer to the diagram from Solution 2. Notice that because , ,
and are parallelograms, , , and .
(Note: I chose to be only because that is what I had written when originally solving.
The solution would work with other choices for .)
Problem 10.
In a parlor game, the magician asks one of the participants to think of a three digit number
(abc) where a, b, and c represent digits in base 10 in the order indicated. The magician then
asks this person to form the numbers (acb), (bca), (bac), (cab), and (cba), to add these five
numbers, and to reveal their sum, . If told the value of , the magician can identify the
original number, (abc). Play the role of the magician and determine the (abc) if .
Solution
Solution 1
Let be the number . Observe that so
Therefore is mod and mod . There is a shared factor in in both, but Chinese
remainder theorem still tells us the value of mod , namely mod . We see
that there are no other 3-digit integers that are mod , so .
Problem 11.
The polynomial may be written in the
form , where and the 's
are constants. Find the value of .
Solution
Solution 1
We want the coefficient of the term of each power of each binomial, which by the binomial
is equal to .
Problem 12.
Let the sum of a set of numbers be the sum of its elements. Let be a set of positive integers,
none greater than 15. Suppose no two disjoint subsets of have the same sum. What is the
largest sum a set with these properties can have?
Solution
The maximum is , attained when . We must now prove that no
such set has sum greater than 61. Suppose such a set existed. Then must have more
than 4 elements, otherwise its sum would be at most .
can't have more than 5 elements. To see why, note that at
Thus, would have to have 5 elements. contains both 15 and 14 (otherwise its sum is at
most ). It follows that cannot contain both and for
any , or the subsets and would have the same sum. So
now must contain 13 (otherwise its sum is at most ),
and cannot contain 12, or the subsets and would have the same sum.
Now the only way could have sum at least would be
if . But the subsets and have the same sum, so this
set does not work. Therefore no with sum greater than 61 is possible and 61 is indeed the
maximum.
Problem 13.
In a sequence of coin tosses, one can keep a record of instances in which a tail is immediately
followed by a head, a head is immediately followed by a head, and etc. We denote these
by TH, HH, and etc. For example, in the sequence HHTTHHHHTHHTTTT of 15 coin tosses we
observe that there are five HH, three HT, two TH, and four TT subsequences. How many
different sequences of 15 coin tosses will contain exactly two HH, three HT, four TH, and
five TT subsequences?
Solution
Let's consider each of the sequences of two coin tosses as an operation instead; this
operation takes a string and adds the next coin toss on (eg, THHTH + HT = THHTHT). We
examine what happens to the last coin toss. Adding HH or TT is simply an identity for the last
coin toss, so we will ignore them for now. However, adding HT or TH switches the last
coin. H switches to T three times, but T switches to H four times; hence it follows that our string
will have a structure of THTHTHTH.
Now we have to count all of the different ways we can add the identities back in. There are
5 TT subsequences, which means that we have to add 5 T into the strings, as long as the
new Ts are adjacent to existing Ts. There are already 4 Ts in the sequence, and since order
doesn’t matter between different tail flips this just becomes the ball-and-urn argument. We
want to add 5 balls into 4 urns, which is the same as 3 dividers; hence this
Problem 14.
The shortest distances between an interior diagonal of a rectangular parallelepiped, , and
the edges it does not meet are , , and . Determine the volume of .
Solution
In the above diagram, we focus on the line that appears closest and is parallel to . All the
blue lines are perpendicular lines to and their other points are on , the main diagonal.
The green lines are projections of the blue lines onto the bottom face; all of the green lines
originate in the corner and reach out to , and have the same lengths as their
corresponding blue lines. So we want to find the shortest distance between and that
corner, which is .
So we have:
Notice the familiar roots: , , , which are , , ,
respectively. (This would give us the guess that the sides are of the ratio 1:2:3, but let's
provide the complete solution.)
Problem 15.
Let triangle be a right triangle in the xy-plane with a right angle at . Given that the
length of the hypotenuse is , and that the medians through and lie along the
lines and respectively, find the area of triangle .
Solution
Translate so the medians are , and , then model
the points and . is the centroid, and is the average of the vertices,
so
so
(1)
AC and BC are perpendicular, so the product of their slopes is -1, giving
(2)
Solution
Since no carrying over is allowed, the range of possible values of any digit of is from to
the respective digit in (the values of are then fixed). Thus, the number of ordered
pairs will be .
If you do not understand the above solution, consider this. For every positive integer , there
is only one whole number that you can add to it to obtain the required sum. Also, the total
number of non-negative integers that are smaller than or equal to an
integer is because there are positive integers that are less than it, in
addition to and itself.
Problem 2.
What is the largest possible distance between two points, one on the sphere of radius 19
with center and the other on the sphere of radius 87 with center ?
Solution
The distance between the two centers of the spheres can be determined via the distance
formula in three
dimensions: .
The largest possible distance would be the sum of the two radii and the distance between the
two centers, making it .
Problem 3.
By a proper divisor of a natural number we mean a positive integral divisor other than 1 and
the number itself. A natural number greater than 1 will be called nice if it is equal to the product
of its distinct proper divisors. What is the sum of the first ten nice numbers?
Solution
Let denote the product of the distinct proper divisors of . A number is nice in one of
two instances:
If we let , where are the prime factors, then its proper divisors are and ,
and .
Alternatively, we could note that is only nice when it only has two divisors, which, when
multiplied, clearly yield . We know that when the prime
factorization of , the number of factors of is
Since is nice, it may only have factors ( , , , and ). This means that . The
number can only be factored into or , which means that
either and , or . Therefore the only two cases are , or .
Problem 4.
Solution
equations: . Thus, .
The maximum and minimum y value is when , which is
when and . Since the graph is symmetric about the y-axis, we just
. Then .
The area of the region enclosed by the graph is that of the quadrilateral defined by the
points . Breaking it up into triangles and solving or
Problem 5.
Find if and are integers such that .
Solution
If we move the term to the left side, it is factorable:
Problem 6.
Rectangle is divided into four parts of equal area by five segments as shown in the
figure, where ,
and is parallel to . Find the length of (in cm) if cm and cm.
Solution
Solution 1
Since , and the areas of the trapezoids and are the
same, then the heights of the trapezoids are the same. Thus both trapezoids have
the trapezoids and are the same, the heights of the trapezoids are both
.From here, we have
Since
, .
Problem 7.
Let denote the least common multiple of positive integers and . Find the number
of ordered triples of positive integers for which , ,
and .
Solution 1
It's clear that we must have , and for
some nonnegative integers . Dealing first with the powers of 2: from the given
conditions, , . Thus we must have and
at least one of equal to 3. This gives 7 possible triples
: and .
Solution 2
and . By looking at the prime factorization of , must have
a factor of . If has a factor of , then there are two cases: either (1) or , or
(2) one of and has a factor of and the other a factor of . For case 1, the other
number will be in the form of , so there are possible such numbers; since this
can be either or there are a total of possibilities. For case 2, and are
in the form of and , with and (if they were equal to 3, it would overlap with
case 1). Thus, there are cases.
If does not have a factor of , then at least one of and must be , and both must
have a factor of . Then, there are solutions possible just considering , and a
total of possibilities. Multiplying by three, as , there are .
Together, that makes solutions for .
Problem 8.
What is the largest positive integer for which there is a unique integer such
that ?
Solution 1
Multiplying out all of the denominators, we get:
Since , . Also, , so .
Thus, . is unique if it is within a maximum range of , so .
Solution 2
Flip all of the fractions for
Continue as in Solution 1.
Problem 9.
Triangle has right angle at , and contains a point for which , ,
and . Find .
Solution
Let . Since , each of them is equal to . By
the Law of Cosines applied to triangles , and at their respective
and
Problem 10.
Al walks down to the bottom of an escalator that is moving up and he counts 150 steps. His
friend, Bob, walks up to the top of the escalator and counts 75 steps. If Al's speed of walking
(in steps per unit time) is three times Bob's walking speed, how many steps are visible on the
escalator at a given time? (Assume that this value is constant.)
Solutions
Solution 1
Let the total number of steps be , the speed of the escalator be and the speed of Bob be .
In the time it took Bob to climb up the escalator he saw 75 steps and also climbed the entire
escalator. Thus the contribution of the escalator must have been an additional steps.
Since Bob and the escalator were both moving at a constant speed over the time it took Bob to
climb, the ratio of their distances covered is the same as the ratio of their speeds,
so .
Similarly, in the time it took Al to walk down the escalator he saw 150 steps, so the escalator
since he moves at a pace of relative to the escalator, it will take time to get
to the top.
at
Solution
Let us write down one such sum, with terms and first term :
Problem 12.
Let be the smallest integer whose cube root is of the form , where is a positive
integer and is a positive real number less than . Find .
Solution
In order to keep as small as possible, we need to make as small as possible.
Problem 13.
A given sequence of distinct real numbers can be put in ascending order by
means of one or more "bubble passes". A bubble pass through a given sequence consists of
comparing the second term with the first term, and exchanging them if and only if the second
term is smaller, then comparing the third term with the second term and exchanging them if
and only if the third term is smaller, and so on in order, through comparing the last term, ,
with its current predecessor and exchanging them if and only if the last term is smaller.
The example below shows how the sequence 1, 9, 8, 7 is transformed into the sequence 1, 8,
7, 9 by one bubble pass. The numbers compared at each step are underlined.
Suppose that , and that the terms of the initial sequence are distinct
from one another and are in random order. Let , in lowest terms, be the probability that the
number that begins as will end up, after one bubble pass, in the place. Find .
Solution
If any of is larger than , one of these numbers will be compared with on the
19th step of the first bubble pass and will be moved back to the 19th position.
Thus, must be the largest of the first 20 terms. In addition, must be larger
than but smaller than in order that it move right to the 30th position but
then not continue moving right to the 31st.
Thus, our problem can be restated: What is the probability that in a sequence of 31 distinct
real numbers, the largest is in position 31 and the second-largest is in position 20 (the other 29
numbers are irrelevant)?
This is much easier to solve: there are ways to order the first thirty-one numbers
and ways to arrange them so that the largest number is in the 31st position and the
Problem 14.
Compute
Solution
The Sophie Germain Identity states that can
be factorized as . Each of the terms is in the form
of . Using Sophie-Germain, we get
that
Solution
Because all the triangles in the figure are similar to triangle , it's a good idea to use area
, . Therefore, .
However, is equal to the area of triangle ! This means that the ratio
between the areas and is , and the ratio between the sides is . As a
result, . We now need to find the value
of , because .
Solution
Currently there are possible combinations. With any integer from to , the number
just .
Note: A simpler way of thinking to get is thinking that each button has two choices, to be
black or
to be white.
Problem 2.
For any positive integer , let denote the square of the sum of the digits of . For ,
let . Find .
Solution
We see that
Solution
Raise both as exponents with base 8:
A quick explanation of the steps: On the 1st step, we use the property
of logarithms that . On the 2nd step, we use the fact that . On
the 3rd step, we use the change of base formula, which states for arbitrary .
Problem 4.
Suppose that for . Suppose further
that What is the smallest possible
value of ?
Solution
Since then
Problem 5.
Let , in lowest terms, be the probability that a randomly chosen positive divisor of is
an integer multiple of . Find .
Solution
, so it has factors. Out of these, we only want
those factors of which are divisible by ; it is easy to draw a bijection to the number of
factors that has, which is . Our probability
is , and .
Problem 6.
It is possible to place positive integers into the vacant twenty-one squares of the square
shown below so that the numbers in each row and column form arithmetic sequences. Find
the number that must occupy the vacant square marked by the asterisk (*).
Solution
Solution 1 (specific)
Let the coordinates of the square at the bottom left be , the square to the right , etc.
Label the leftmost column (from bottom to top) and the bottom-most row (from
left to right) . Our method will be to use the given numbers to set up equations to
solve for and , and then calculate .
We can compute the squares at the intersections of two existing numbers in terms of and ;
two such equations will give us the values of and . On the fourth row from the bottom, the
common difference is , so the square at has a value of . On the third
column from the left, the common difference is , so that square also has a value
of . Equating, we
get .
Now we compute the square . By rows, this value is simply the average of and ,
Solving
Now, has , so .
Solution 2 (general)
First, let the number to be placed in the first column, fourth row. Let the number to be
placed in the second column, fifth row. We can determine the entire first column and fifth row
in terms of and :
Next, let the number to be placed in the second column, fourth row. We can
determine the entire second column and fourth row in terms of , , and :
We have now determined at least two values in each row and column. We can finish the table
without introducing any more variables:
Problem 7.
In triangle , , and the altitude from divides into segments of
length 3 and 17. What is the area of triangle ?
Solution
Let be the intersection of the altitude with , and be the length of the altitude. Without
loss of generality, let and .
Problem 8.
The function , defined on the set of ordered pairs of positive integers, satisfies the following
properties: Calculate .
Solution
Since all of the function's properties contain a recursive definition except for the first one, we
know that in order to obtain an integer answer. So, we have to
transform to this form by exploiting the other properties. The second one doesn't
help us immediately, so we will use the third one.
Note that
.
Hence, is a solution to the functional equation.
Since this is an AIME problem, there is exactly one correct answer, and thus, exactly one
possible value of .
Therefore, .
Problem 9.
Find the smallest positive integer whose cube ends in .
Solution
A little bit of checking tells us that the units digit must be 2. Now our cube must be in the form
of ; using the binomial theorem gives us . Since we
are looking for the tens digit, we get . This is true if the
tens digit is either or . Casework:
Problem 10.
A convex polyhedron has for its faces 12 squares, 8 regular hexagons, and 6
regular octagons. At each vertex of the polyhedron one square, one hexagon, and one
octagon meet. How many segments joining vertices of the polyhedron lie in the interior of the
polyhedron rather than along an edge or a face?
Solution 1
Solution 2
We first find the number of vertices on the polyhedron: There are 4 corners per square, 6
corners per hexagon, and 8 corners per octagon. Each vertex is where 3 corners coincide, so
the diagram. You can draw a diagram to count this better: Since 13 aren't
possible endpoints, that means that there are 35 possible endpoints per vertex. The total
number of segments joining vertices that aren't on the same face
is
Solution 3
Since at each vertex one square, one hexagon, and one octagon meet, then there are a total
of vertices. This means that for each segment we
have choices of vertices for the first endpoint of the segment.
Since each vertex is the meeting point of a square, octagon, and hexagon, then there
are other vertices of the square that are not the first one, and connecting the first point to any
of these would result in a segment that lies on a face or edge.
Similarly, there are points on the adjacent hexagon and points on adjoining octagon that,
when connected to the first point, would result in a diagonal or edge.
However, the square and hexagon share a vertex, as do the square and octagon, and the
hexagon and octagon.
Subtracting these from the vertices we have left to choose from, and adding the that we
counted twice, we get
Problem 11.
Let be complex numbers. A line in the complex plane is called a
mean line for the points if contains points (complex
Solution
Solution 2
We know that
And because the sum of the 5 's must cancel this out,
We write the numbers in the form and we know that
and
The line is of equation . Substituting in the polar coordinates, we
have .
Summing all 5 of the equations given for each , we get
Problem 12.
Let be an interior point of triangle and extend lines from the vertices through to the
opposite sides. Let , , , and denote the lengths of the segments indicated in the figure.
Find the product if and .
Solution 1
Call the cevians AD, BE, and CF. Using area ratios ( and have the same
base), we have:
Similarily, and .
Then,
Solution 2
Let be the weights of the respective vertices. We see that the weights of the
feet of the cevians are . By mass points, we have that:
If we add the equations together, we
get
If we multiply them together, we
get
Problem 13.
Find if and are integers such that is a factor of .
Solution 1
Let's work backwards! Let and let be the polynomial such
that .
Clearly, the constant term of must be . Now, we
have , where is some coefficient.
However, since has no term, it must be true that .
Let's find now. Notice that all we care about in finding is
that .
Therefore, . Undergoing a similar process, , , , and we see
a nice pattern. The coefficients of are just the Fibonacci sequence with alternating signs!
Therefore, , where denotes the 16th Fibonnaci number and .
Solution 2
Let represent the th number in the Fibonacci sequence. Therefore,
and
Solution 3
We can long divide and search for a pattern; then the remainder would be set to zero to solve
for . Writing out a few examples quickly shows us that the remainders after each subtraction
follow the Fibonacci sequence. Carrying out this pattern, we find that the remainder
is . Since the coefficient of must be zero, this
gives us two equations, and . Solving these two as
above, we get that .
There are various similar solutions which yield the same pattern, such as repeated substitution
of into the larger polynomial.
Solution 4
The roots of are (the Golden Ratio) and . These two must also be roots
of . Thus, we have two
equations: and . Subtract these two
and divide by to get . Noting that the
Problem 14.
Let be the graph of , and denote by the reflection of in the line . Let
the equation of be written in the form
Solution 1
Given a point on , we look to find a formula for on . Both points lie on
Therefore .
Thus, .
Solution 2
The asymptotes of are given by and . Now if we represent the line by
the complex number , then we find the direction of the reflection of the
asymptote by multiplying this by , getting . Therefore, the asymptotes
of are given by and .
Now to find the equation of the hyperbola, we multiply the two expressions together to get one
side of the equation: . At this point, the right
hand side of the equation will be determined by plugging the point , which is
unchanged by the reflection, into the expression. But this is not necessary. We see
that , , so .
Solution 3
The matrix for a reflection about the polar line is: This
is not hard to derive using a basic knowledge of linear transformations. You can refer here for
more information: https://en.wikipedia.org/wiki/Orthogonal_matrix
Let . Note that the line of reflection, , is the polar line .
Then , so and .
Therefore, if is mapped to under the reflection,
Problem 15.
In an office at various times during the day, the boss gives the secretary a letter to type, each
time putting the letter on top of the pile in the secretary's in-box. When there is time, the
secretary takes the top letter off the pile and types it. There are nine letters to be typed during
the day, and the boss delivers them in the order .
While leaving for lunch, the secretary tells a colleague that letter has already been typed, but
says nothing else about the morning's typing. The colleague wonders which of the nine letters
remain to be typed after lunch and in what order they will be typed. Based upon the above
information, how many such after-lunch typing orders are possible? (That there are no letters
left to be typed is one of the possibilities.)
Solution
Re-stating the problem for clarity, let be a set arranged in increasing order. At any time an
element can be appended to the end of , or the last element of can be removed. The
question asks for the number of different orders in which the all of the remaining elements
of can be removed, given that had been removed already.
Since had already been added to the pile, the numbers had already been added at
some time to the pile; might or might not have been added yet. So currently is a subset
of , possibly with at the end. Given that has elements, there
are intervals for to be inserted, or might have already been placed,
giving different possibilities.
Thus, the answer
is
.
AIME 1989
Problem 1.
Compute .
Solution
Solution 1
. .
Solution 3
The last digit under the radical is , so the square root must either end in or ,
since means . Additionally, the number must be
near , narrowing the reasonable choices to and .
Continuing the logic, the next-to-last digit under the radical is the same as the last digit
of , which is . Quick computation shows that ends in , while ends
in . Thus, the answer is .
Solution 4
Similar to Solution 1 above, call the consecutive
It's equal to , which simplifies to . You can plug in the value of from
Problem 2.
Ten points are marked on a circle. How many distinct convex polygons of three or more sides
can be drawn using some (or all) of the ten points asvertices?
Solution
Any subset of the ten points with three or more members can be made into exactly one such
polygon. Thus, we need to count the number of such subsets. There are total
Note is equivalent to
Problem 3.
Suppose is a positive integer and is a single digit in base 10. Find if
Solution
Repeating decimals represent rational numbers. To figure out which rational number, we sum
Solution 2
To get rid of repeating decimals, we multiply the equation by 1000. We
Problem 4.
If are consecutive positive integers such that is a perfect
square and is a perfect cube, what is the smallest possible value of ?
Solution
Since the middle term of an arithmetic progression with an odd number of terms is the average
of the series, we know and . Thus, must be in the
form of based upon the first part and in the form of based upon the second part,
with and denoting an integers. is minimized if it’s prime factorization contains only ,
and since there is a cubed term in , must be a factor of . , which works
as the solution.
Problem 5.
When a certain biased coin is flipped five times, the probability of getting heads exactly once is
not equal to and is the same as that of getting heads exactly twice. Let , in lowest terms, be
the probability that the coin comes up heads in exactly out of flips. Find .
Solution
Solution 1
Denote the probability of getting a heads in one flip of the biased coin as . Based upon the
is , so .
Solution 2
Denote the probability of getting a heads in one flip of the biased coins as and the probability
simplifying is , so .
Problem 6.
Two skaters, Allie and Billie, are at points and , respectively, on a flat, frozen lake.
The distance between and is meters. Allie leaves and skates at
a speed of meters per second on a straight line that makes a angle with . At the
same time Allie leaves , Billie leaves at a speed of meters per second and follows
the straight path that produces the earliest possible meeting of the two skaters, given their
speeds. How many meters does Allie skate before meeting Billie?
Solution
Label the point of intersection as . Since , and . According to
the law of cosines,
Since we are looking for the earliest possible intersection, seconds are needed.
Thus, meters is the solution.
Problem 7.
If the integer is added to each of the numbers , , and , one obtains the squares of
three consecutive terms of an arithmetic series. Find .
Solution
Call the terms of the arithmetic progression , making their
squares .
We know that and , and subtracting these two we
get (1). Similarly, using and ,
subtraction yields (2).
Subtracting the first equation from the second, we get , so . Substituting
backwards yields that and .
Problem 8.
Assume that are real numbers such that
Solution
Solution 1
Notice that because we are given a system of equations with unknowns, the
values are not fixed; indeed one can take any four of the variables and
assign them arbitrary values, which will in turn fix the last three.
Given this, we suspect there is a way to derive the last expression as a linear combination of
the three given expressions. Let the coefficent of in the first equation be ; then its
coefficients in the second equation is and the third as . We need to find a
way to sum these to make [this is in fact a specific approach generalized by the next
solution below].
Thus, we hope to find
constants satisfying . FOILing out all of
the terms, we get
Subtracting the second and third equations yields that , so and . It follows
that the desired expression is .
Solution 2
Notice that we may rewrite the equations in the more compact form as:
and
where and is what we're trying to find.
Now consider the polynomial given by (we are only treating the as
coefficients).
Notice that is in fact a quadratic. We are given as and are
asked to find . Using the concept of finite differences(a prototype of differentiation) we find
that the second differences of consecutive values is constant, so that by arithmetic operations
we find .
obtains .
Solution 3
Notice that
I'll number the equations for convenience
So
Solution 4
Notice subracting the first equation from the second
yields Then, repeating for the 2nd and 3rd equations, and then
subtracting the result from the first obtained equation, we get .
Adding this twice to the first obtained equation gives difference of the desired equation and 3rd
equation, which is 211. Adding to the 3rd equation, we get
Problem 9.
One of Euler's conjectures was disproved in the 1960s by three American mathematicians
when they showed there was a positive integer such that .
Find the value of .
Solution 1
Note that is even, since the consists of two odd and two even numbers. By Fermat's
Little Theorem, we know is congruent to modulo5. Hence,
Solution 2
We can cheat a little bit and approximate, since we are dealing with such large numbers. As
above, , and it is easy to see that .
Therefore, , so the last digit
of is 4.
We notice that and are all very close or equal to multiples of 27. We can
rewrite as approximately equal to . This
134 will obviously be too small, so we try 144. . Bashing through the
division, we find that , which is very close to . It is clear that 154 will
not give any closer of an answer, given the rate that fifth powers grow, so we can safely
assume that is the answer.
Problem 10.
Let , , be the three sides of a triangle, and let , , , be the angles opposite them.
If , find
Solution
Solution 1
We can draw the altitude to , to get two right triangles. , from the
Now
Solution 3
Next, we are going to put all the sin's in term of . We get . Therefore,
we get .
Next, use Law of Cosines to give us .
get .
Solution 4
Let be
WLOG, assume that and are legs of right triangle with and
By Pythagorean theorem, we have , and the given . Solving the
equations gives us and . We see that , and
.
We see that our derived equation equals to as approaches infinity.
Evaluating , we get
Problem 11.
A sample of 121 integers is given, each between 1 and 1000 inclusive, with repetitions
allowed. The sample has a unique mode (most frequent value). Let be the difference
between the mode and the arithmetic mean of the sample. What is the largest possible value
of ? (For real , is the greatest integer less than or equal to .)
Solution
Let the mode be , which we let appear times. We let the arithmetic mean be , and
the sum of the numbers be . Then
As is essentially independent
of , it follows that we wish to minimize or maximize (in other words, ).
Indeed, is symmetric about ; consider replacing all of numbers in the
sample with , and the value of remains the same. So, without loss of generality,
let . Now, we would like to maximize the quantity
of . We let , so
Expanding (ignoring the constants, as these do not affect which yields a maximum) and
In less formal language, it quickly becomes clear after some trial and error that in our sample,
there will be values equal to one and values each of . It is fairly
easy to find the maximum. Try , which yields , , which yields , ,
which yields , and , which yields . The maximum difference occurred at ,
so the answer is .
Notes
^ In fact, when (which some simple testing shows that the maximum will
indeed .
Problem 12.
Let be a tetrahedron with , , , , ,
and , as shown in the figure. Let be the distance between
the midpoints of edges and . Find .
Solution
Call the midpoint of and the midpoint of . is the median of triangle .
The formula for the length of a median is , where , , and are the
side lengths of triangle, and is the side that is bisected by median . The formula is a direct
result of the Law of Cosines applied twice with the angles formed by the median (Stewart's
Theorem).
We first find , which is the median of .
Solution
We first show that we can choose at most 5 numbers from such that no two
numbers have a difference of or . We take the smallest number to be , which rules out
. Now we can take at most one from each of the pairs: , , , .
Now, . Because this isn't an exact multiple of , we need to consider
some numbers separately.
Notice that . Therefore we can put the last numbers
into groups of 11. Now let's examine . If we pick from the
first numbers, then we're allowed to pick , , , , . This
means we get 10 members from the 20 numbers. Our answer is thus .
Problem 14.
Given a positive integer , it can be shown that every complex number of the form ,
where and are integers, can be uniquely expressed in the base using the
integers as digits. That is, the equation
is true for a unique choice of non-negative integer and digits chosen from
the set , with . We write
Solution
First, we find the first three powers of :
Case 1:
Case 2:
Problem 15.
Point is inside . Line segments , , and are drawn with on
, on , and on (see the figure below). Given that , ,
, , and , find the area of .
Solution
Solution 1
Because we're given three concurrent cevians and their lengths, it seems very tempting to
apply Mass points. We immediately see that , , and . Now, we
recall that the masses on the three sides of the triangle must be balanced out,
so and . Thus, and .
Recalling that , we see that and is
a median to in . Applying Stewart's Theorem, ,
and . Now notice that , because both triangles share the
same base and the . Applying Heron's formula on triangle with
sides , , and , and .
Solution 2
Using a different form of Ceva's Theorem, we
have
Solving and , we obtain and .
Let be the point on such that . Since and
, . (Stewart's Theorem)
thus .
is a right triangle, so ( ) is . Therefore, the area
of . Using area
ratio, .
Solution 3
Because the length of cevian is unknown, we can examine what happens when we
extend it or decrease its length and see that it simply changes the angles between the
cevians. Wouldn't it be great if it the length of was such that ? Let's first
assume it's a right angle and hope that everything works out.
Extend to so that . The result is that , ,
and because . Now we see that if we are able to show
that , that is , then our right angle assumption will be true.
Apply the Pythagorean Theorem on to get ,
so and . Now, we apply the Law of Cosines on
triangles and .
Let . Notice that and , so we
get two nice equations.
Solving, (yay!).
Now, the area is easy to
find. .
AIME 1990
Problem 1.
The increasing sequence consists of all positive integers that are
neither the square nor the cube of a positive integer. Find the 500th term of this sequence.
Solution
Because there aren't that many perfect squares or cubes, let's look for the smallest perfect
square greater than . This happens to be . Notice that there are squares
and cubes less than or equal to , but and are both squares and cubes. Thus, there
are numbers in our sequence less than . Magically, we want
the term, so our answer is the smallest non-square and non-cube less than , which
is .
Problem 2.
Find the value of .
Solution
Suppose that is in the form of . FOILing yields
that . This implies that and equal one of .
The possible sets are and ; the latter can be discarded since the square
root must be positive. This means that . Repeating this
for , the only feasible possibility is .
Problem 3.
Let be a regular and be a regular such that each interior
angle of is as large as each interior angle of . What's the largest possible value of ?
Solution
get .
and , making the numerator of the fraction positive. To make
the denominator positive, ; the largest possible value of is .
This is achievable because the denominator is , making a positive
number and .
Problem 4.
Find the positive solution to
Solution
We could clear out the denominators by multiplying, though that would be unnecessarily
tedious.
To simplify the equation, substitute (the denominator of the first fraction).
Simplifying, , so . Re-
substituting, . The positive root is .
Problem 5.
Let be the smallest positive integer that is a multiple of and has exactly positive
Solution
The prime factorization of . For to have
exactly integral divisors, we need to have such that .
Since , two of the prime factors must be and . To minimize , we can introduce a third
prime factor, . Also to minimize , we want , the greatest of all the factors, to be raised to
Problem 6.
A biologist wants to calculate the number of fish in a lake. On May 1 she catches a random
sample of 60 fish, tags them, and releases them. On September 1 she catches a random
sample of 70 fish and finds that 3 of them are tagged. To calculate the number of fish in the
lake on May 1, she assumes that 25% of these fish are no longer in the lake on September 1
(because of death and emigrations), that 40% of the fish were not in the lake May 1 (because
of births and immigrations), and that the number of untagged fish and tagged fish in the
September 1 sample are representative of the total population. What does the biologist
calculate for the number of fish in the lake on May 1?
Solution 1
Of the fish caught in September, were not there in May, so fish were there in May.
Since the percentage of tagged fish in September is proportional to the percentage of tagged
fish in May, .
(Note the 25% death rate does not affect the answer because both tagged and nontagged fish
die.)
Solution 2
First, we notice that there are 45 tags left, after 25% of the original fish have went away/died.
Then, some percent of fish have been added such that , or . Solving for ,
we get that , so the total number of fish in September is , or times the total
number of fish in May.
Since of the fish in September were tagged, , where is the number of fish in
May. Solving for , we see that
Problem 7.
A triangle has vertices , , and . The equation of
the bisector of can be written in the form . Find .
Solution
Use the distance formula to determine the lengths of each of the sides of the triangle. We find
that it has lengths of side , indicating that it is a right triangle. At this
point, we just need to find another point that lies on the bisector of .
Solution 1
Use the angle bisector theorem to find that the angle bisector of divides into
so .
The desired answer is the equation of the line . has slope , from which we find
the equation to be . Therefore, .
Solution 2
Extend to a point such that . This forms an isosceles triangle .
The coordinates of , using the slope of (which is ), can be determined to
be . Since the angle bisector of must touch the midpoint of ,
we have found our two points. We reach the same answer of .
Solution 3
By the angle bisector theorem as in solution 1, we find that . If we draw the right
triangle formed by and the point directly to the right of and below , we get
another (since the slope of is ). Using this, we find that the horizontal
projection of is and the vertical projection of is .
Problem 8.
In a shooting match, eight clay targets are arranged in two hanging columns of three targets
each and one column of two targets. A marksman is to break all the targets according to the
following rules:
1) The marksman first chooses a column from which a target is to be broken.
2) The marksman must then break the lowest remaining target in the chosen column.
If the rules are followed, in how many different orders can the eight targets be broken?
Solution
Suppose that the columns are labeled , , and . Consider the string .
Since the arrangements of the strings is bijective to the order of shooting, the answer is the
Problem 9.
A fair coin is to be tossed times. Let , in lowest terms, be the probability that heads
never occur on consecutive tosses. Find .
Solution
Solution 1
Clearly, at least tails must be flipped; any less, then by the Pigeonhole Principle there will be
heads that appear on consecutive tosses.
Consider the case when tails occur. The heads must fall between the tails such that no two
heads fall between the same tails, and must fall in the positions labeled :
There are six slots for the heads to be placed, but only heads remaining. Thus,
are . There
are a total of possible flips of coins, making the probability . Thus, our
solution is .
Solution 2
Call the number of ways of flipping coins and not receiving any consecutive heads .
Notice that tails must be received in at least one of the first two flips.
If the first coin flipped is a T, then the remaining flips must fall under one of the
configurations of .
If the first coin flipped is a H, then the second coin must be a T. There are
then configurations.
Thus, . By counting, we can establish that and .
Therefore, , forming the Fibonacci sequence. Listing them out, we
get , and the 10th number is . Putting this over to
Problem 10.
The sets and are both sets of complex roots of
unity. The set is also a set of complex roots of unity. How
many distinct elements are in ?
Solution
Solution 1
The least common multiple of and is , so define . We can write the
numbers of set as and of set as . can yield at
most different values. All solutions for will be in the form of .
and are relatively prime, and it is well known that for two relatively prime integers , the
largest number that cannot be expressed as the sum of multiples of is .
For , this is ; however, we can easily see that the numbers to can be written in
terms of . Since the exponents are of roots of unities, they reduce , so all
numbers in the range are covered. Thus the answer is .
Solution 2
The 18 and 48th roots of can be found by De Moivre's Theorem. They
Problem 11.
Someone observed that . Find the largest positive integer for which can be
expressed as the product of consecutivepositive integers.
Solution 1
The product of consecutive integers can be written as for some integer .
as .
For , we get so . For greater values of , we need to find the product
of consecutive integers that equals . can be approximated as , which
decreases as increases. Thus, is the greatest possible value to satisfy the given
conditions.
Solution 2
Let the largest of the consecutive positive integers be . Clearly cannot be less than
or equal to , else the product of consecutive positive integers will be less than .
Key observation: Now for to be maximum the smallest number (or starting number) of
the consecutive positive integers must be minimum, implying that needs to be
minimum. But the least is .
So the consecutive positive integers are
So we have
Generalization:
Largest positive integer for which can be expressed as the product of consecutive
positive integers is
For ex. largest such that product of consecutive positive integers is equal
to is
Proof: Reasoning the same way as above, let the largest of the consecutive positive
integers be . Clearly cannot be less than or equal to , else the product
of consecutive positive integers will be less than .
Now, observe that for to be maximum the smallest number (or starting number) of
the consecutive positive integers must be minimum, implying that needs to be
minimum. But the least is .
So the consecutive positive integers are
So we have
Problem 12.
A regular 12-gon is inscribed in a circle of radius 12. The sum of the lengths of all sides
and diagonals of the 12-gon can be written in the form where , ,
, and are positive integers. Find .
Solution 1
The easiest way to do this seems to be to find the length of each of the sides and diagonals.
To do such, draw the radii that meet the endpoints of the sides/diagonals; this will
form isosceles triangles. Drawing the altitude of those triangles and then solving will yield the
respective lengths.
. .
The length of each of the 12 diagonals that span across 2 edges is (or
notice that the triangle formed is equilateral).
The length of each of the 12 diagonals that span across 3 edges
is (or notice that the triangle formed is a right
triangle).
The length of each of the 12 diagonals that span across 4 edges is .
The length of each of the 12 diagonals that span across 5 edges
is .
The length of each of the 6 diameters is .
. Thus,
the answer is .
Solution 2
A second method involves drawing a triangle connecting the center of the 12-gon to two
vertices of the 12-gon. Since the distance from the center to a vertex of the 12-gon is , the
Law of Cosines can be applied to this isosceles triangle, to give:
To simplify the two nested radicals, add them, and call the sum :
Squaring both sides, the F and L part of FOIL causes the radicals to cancel, leaving :
Problem 13.
Let . Given that has 3817 digits and that its
first (leftmost) digit is 9, how many elements of have 9 as their leftmost digit?
Solution
When a number is multiplied by , it gains a digit unless the new number starts with a 9.
Problem 14.
The rectangle below has dimensions and
. Diagonals and intersect at . If triangle is cut out and removed,
edges and are joined, and the figure is then creased along segments and ,
we obtain a triangular pyramid, all four of whose faces are isosceles triangles. Find the volume
of this pyramid.
Solution
Solution 1
let be the center of a sphere with radius , then lie on the sphere. The cross
section of the sphere that contains is a circle, and the center of that circle is the foot
of the perpendicular from the center of the sphere. Hence the foot of the height we want to find
occurs at the circumcenter of .
From here we just need to perform some brutish calculations. Using the
points , , and is , we can write three equations using the distance formula:
Subtracting the last two equations, we get . Solving for with a bit of effort, we
is .
Problem 15.
Find if the real numbers , , , and satisfy the equations
Solution 1
Set and . Then the relationship
can be exploited:
Therefore:
Solution 2
A recurrence of the form will have the closed form ,
where are the values of the starting term that make the sequence geometric, and are
the appropriately chosen constants such that those special starting terms linearly combine to
form the actual starting terms.
Suppose we have such a recurrence with and .
Then , and .
Solving these simultaneous equations for and , we see that and .
So, .
AIME 1991
Problem 1.
Find if and are positive integers such that
Solution
Solution 1
Define and . Then and . Solving these two equations
yields a quadratic: , which factors to .
Either and or and . For the first case, it is easy to see
that can be (or vice versa). In the second case, since all factors of must
be , no two factors of can sum greater than , and so there are no integral solutions
for . The solution is .
Solution 2
Since , this can be factored to .
As and are integers, the possible sets for (ignoring cases where since it is
symmetrical) are . The second equation factors
to . The only set with a factor of is , and checking
shows that it is our solution.
Problem 2.
Rectangle has sides of length 4 and of length 3. Divide into
168 congruent segments with points , and divide into 168
congruent segments with points . For , draw the
segments . Repeat this construction on the sides and , and then draw
the diagonal . Find the sum of the lengths of the 335 parallel segments drawn.
Solution
The length of the diagonal is (a 3-4-5 right triangle). For each , is
diagonal.
Problem 3.
Expanding by the binomial theorem and doing no further manipulation gives
where for .
For which is the largest?
Solution
Solution 1
looking for must satisfy , where denotes the greatest integer less than
or equal to .
In summary, substituting and we finally find that .
Solution 2
We know that once we have found the largest value of , all values after are less than .
Therefore, we are looking for the smallest possible value such that:
Dividing by gives:
.
We can express these binomial coefficients as factorials.
Therefore, since this identity holds for all values of , the largest possible value
of is .
Solution 3
We know that will increase as increases until certain ,
where and
Problem 4.
Solution
The range of the sine function is . It is periodic (in this problem) with a period of .
Problem 5.
Given a rational number, write it as a fraction in lowest terms and calculate the product of the
resulting numerator and denominator. For how many rational numbers between 0 and 1
will be the resulting product?
Solution
If the fraction is in the form , then and . There are 8 prime numbers less
than 20 ( ), and each can only be a factor of one of or . There
are ways of selecting some combination of numbers for ; however, since , only half
Problem 6.
Suppose is a real number for which
Solution
There are numbers in the sequence. Since the terms of the sequence can
be at most apart, all of the numbers in the sequence can take one of two possible values.
Since , the values of each of the terms of the sequence must be either or . As
the remainder is , must take on of the values, with being the value of the
Problem 7.
Find , where is the sum of the absolute values of all roots of the following equation:
Solution
form on the right hand side, which makes the equation simplify to a quadratic. As this
quadratic will have two roots, they must be the same roots as the quadratic .
The given finite expansion can then be easily seen to reduce to the quadratic
Solution 2
Problem 8.
For how many real numbers does the quadratic equation have only
integer roots for ?
Solution
Solution 1
By Vieta's formulas, where are the roots of the quadratic, and
since are integers, must be an integer. Applying the quadratic formula,
; since is the average of each respective pair and is also an integer, the pairs that work must
have the same parity. Thus we get pairs (counting positive and negative) of factors that
work, and substituting them backwards show that they all work.
Solution 2
Let . Vieta's yields .
Problem 9.
Solution
Solution 1
Use the two trigonometric Pythagorean
identities and .
together and dividing by 2 gives , and subtracting the equations and dividing
by 2 gives .
Hence, and .
Thus, and . Finally,
so .
Solution 3 (least computation)
Subtracting both of the two given equations from this, and simpliyfing with the
identity , we get
calculus). , so
, or
equivalently,
just
Problem 10.
Two three-letter strings, and , are transmitted electronically. Each string is sent letter
by letter. Due to faulty equipment, each of the six letters has a 1/3 chance of being received
incorrectly, as an when it should have been a , or as a when it should be an . However,
whether a given letter is received correctly or incorrectly is independent of the reception of any
other letter. Let be the three-letter string received when is transmitted and let be the
three-letter string received when is transmitted. Let be the probability that comes
before in alphabetical order. When is written as a fraction in lowest terms, what is
its numerator?
Solution
Solution 1
Let us make a chart of values in alphabetical order, where are the probabilities that
each string comes from and multiplied by , and denotes the partial
recursion: .
Problem 11.
Twelve congruent disks are placed on a circle of radius 1 in such a way that the twelve
disks cover , no two of the disks overlap, and so that each of the twelve disks is tangent to
its two neighbors. The resulting arrangement of disks is shown in the figure below. The sum of
the areas of the twelve disks can be written in the from , where are positive
integers and is not divisible by the square of any prime. Find .
Solution
We wish to find the radius of one circle, so that we can find the total area.
Notice that for them to contain the entire circle, each pair of circles must be tangent on the
larger circle. Now consider two adjacent smaller circles. This means that the line connecting
the radii is a segment of length that is tangent to the larger circle at the midpoint of the two
centers. Thus, we have essentially have a regular dodecagon whose vertices are the centers
of the smaller triangles circumscribed about a circle of radius .
We thus know that the apothem of the dodecagon is equal to . To find the side length, we
make a triangle consisting of a vertex, the midpoint of a side, and the center of the dodecagon,
which we denote and respectively. Notice that , and that is a right
triangle with hypotenuse and . Thus ,
which is the radius of one of the circles. The area of one circle is
thus , so the area of all circles is , giving an
answer of .
Problem 11.
A drawer contains a mixture of red socks and blue socks, at most in all. It so happens
that, when two socks are selected randomly without replacement, there is a probability of
exactly that both are red or both are blue. What is the largest possible number of red socks
in the drawer that is consistent with this data?
Solution
Solution 1
Let and denote the number of red and blue socks, respectively. Also, let . The
probability that when two socks are drawn randomly, without replacement, both are red or
both are blue is given by
Now, since and are positive integers, it must be the case that , with .
Hence, would correspond to the general solution. For the present
case , and so one easily finds that is the largest possible integer satisfying
the problem conditions.
to
is .
Solution 3
Let and denote the number of red and blue socks, respectively. In addition, let ,
the total number of socks in the drawer.
Expanding, we get
Substituting for and cross multiplying, we
get
Combining terms, we get
To make this expression factorable, we add to both sides, resulting
in
From this equation, we can test values for the expression , which is the
multiplication of two consecutive integers, until we find the highest value of or such
that .
By testing and , we get that and
. Testing values one integer higher, we get that and .
Since is greater than , we conclude that is our answer.
Since it doesn't matter whether the number of blue or red socks is , we take the higher
value for , thus the maximum number of red socks is .
Solution 4
As above, let , , and denote the number of red socks, the number of blue socks, and the
so .
Seeing that we can rewrite as , and remembering that , we
which is . Therefore, .
Solution by Zeroman
Problem 12.
Rhombus is inscribed in rectangle so that vertices , , , and are
interior points on sides , , , and , respectively. It is given that
, , , and . Let , in lowest terms, denote
the perimeter of . Find .
Solution
Solution 1
Let be the center of the rhombus. Via parallel sides and alternate interior angles, we see
that the opposite triangles are congruent ( , ). Quickly
we realize that is also the center of the rectangle.
By the Pythagorean Theorem, we can solve for a side of the
rhombus; . Since the diagonals of a rhombus are perpendicular
bisectors, we have that . Also, , so
quadrilateral is cyclic. By Ptolemy's Theorem, .
By similar logic, we have is a cyclic quadrilateral. Let , . The
Pythagorean Theorem gives us . Ptolemy’s Theorem gives
We reject because then everything degenerates into squares, but the condition
rectangle is
Solution 3
The triangles are isosceles, and similar (because they have
).
Hence .
The length of could be found easily from the area of :
From the right triangle we have . We could
have also defined a similar formula: , and then we found , the
segment is tangent to the circles with diameters .
The perimeter
is .
Solution 4
For convenience, let . Since the opposite triangles are congruent we have
that , and therefore . Let , then we
Problem 13.
A drawer contains a mixture of red socks and blue socks, at most in all. It so happens
that, when two socks are selected randomly without replacement, there is a probability of
exactly that both are red or both are blue. What is the largest possible number of red socks
in the drawer that is consistent with this data?
Solution
Solution 1
Let and denote the number of red and blue socks, respectively. Also, let . The
probability that when two socks are drawn randomly, without replacement, both are red or
both are blue is given by
to
is .
Solution 3
Let and denote the number of red and blue socks, respectively. In addition, let ,
the total number of socks in the drawer.
Expanding, we get
Substituting for and cross multiplying, we
get
Combining terms, we get
To make this expression factorable, we add to both sides, resulting
in
From this equation, we can test values for the expression , which is the
multiplication of two consecutive integers, until we find the highest value of or such
that .
By testing and , we get that and
. Testing values one integer higher, we get that and .
Since is greater than , we conclude that is our answer.
Since it doesn't matter whether the number of blue or red socks is , we take the higher
value for , thus the maximum number of red socks is .
Solution 4
As above, let , , and denote the number of red socks, the number of blue socks, and the
so .
Seeing that we can rewrite as , and remembering that , we
which is . Therefore, .
Solution by Zeroman
Problem 14.
A hexagon is inscribed in a circle. Five of the sides have length and the sixth, denoted
by , has length . Find the sum of the lengths of the three diagonals that can be drawn
from .
Solution
Let , , and .
Ptolemy's Theorem on gives , and Ptolemy
on gives . Subtracting these equations
give , and from this . Ptolemy
on gives , and from this . Finally, plugging back into the first
equation gives , so .
Problem 15.
For positive integer , define to be the minimum value of the
Note that
Solution
Solution 1
There are 8 fractions which fit the conditions between 0 and
1:
Their sum is 4. Note that there are also 8 terms between 1 and 2 which we can obtain by
adding 1 to each of our first 8 terms. For example, Following this pattern, our
answer is
Solution 2
By Euler's Totient Function, there are numbers that are relatively prime to , less than .
Note that they come in pairs which result in sums of ; thus the sum of the
Problem 2.
A positive integer is called ascending if, in its decimal representation, there are at least two
digits and each digit is less than any digit to its right. How many ascending positive integers
are there?
Solution
Note that an ascending number is exactly determined by its digits: for any set of digits (not
including 0, since the only position for 0 is at the leftmost end of the number, i.e. a leading 0),
there is exactly one ascending number with those digits.
So, there are nine digits that may be used: Note that each digit may be
present or may not be present. Hence, there are potential ascending numbers, one
for each subset of .
However, we've counted one-digit numbers and the empty set, so we must subtract them off to
get our answer,
Problem 3.
A tennis player computes her win ratio by dividing the number of matches she has won by the
total number of matches she has played. At the start of a weekend, her win ratio is
exactly . During the weekend, she plays four matches, winning three and losing one. At
the end of the weekend, her win ratio is greater than . What's the largest number of
matches she could've won before the weekend began?
Solution
Let be the number of matches won, so that , and .
Problem 4.
In Pascal's Triangle, each entry is the sum of the two entries above it. In which row of Pascal's
Triangle do three consecutive entries occur that are in the ratio ?
Solution
In Pascal's Triangle, we know that the binomial coefficients of the th row
are . Let our row be the th row such that the three consecutive entries
are , and .
After expanding and dividing one entry by another (to clean up the factorials), we see
Problem 5.
Let be the set of all rational numbers , , that have a repeating decimal
expansion in the form , where the digits , , and are not
necessarily distinct. To write the elements of as fractions in lowest terms, how many
different numerators are required?
Solution
We consider the method in which repeating decimals are normally converted to fractions with
an example:
Thus, let
If is not divisible by or , then this is in lowest terms. Let us consider the other
multiples: multiples of , of , and of and , so ,
which is the amount that are neither. The numbers that are multiples of reduce to
multiples of . We have to count these since it will reduce to a multiple of which we have
removed from , but, this cannot be removed since the numerator cannot cancel the
.There aren't any numbers which are multiples of , so we can't get numerators which are
multiples of . Therefore .
Problem 6.
For how many pairs of consecutive integers in is no carrying
required when the two integers are added?
Solution
Solution 1
Consider what carrying means: If carrying is needed to add two numbers with
digits and , then or or . 6.
Consider . has no carry if . This
gives possible solutions.
With , there obviously must be a carry. Consider
. have no carry. This gives possible solutions.
Considering , have no carry. Thus, the solution
is .
Solution 2
Consider the ordered pair where and are digits. We are trying to find
all ordered pairs where does not require carrying. For the addition to
require no carrying, , so unless ends in , which we will address
later. Clearly, if , then adding will require no
carrying. We have possibilities for the value of , for , and for , giving a total
of , but we are not done yet.
We now have to consider the cases where , specifically
when . We can see
that , and all work, giving a grand total
of ordered pairs.
Solution 3
There are 3 forms possible. : , : , : , : --- Thus,
since there should be no carrying, in only integers to is possible Therefore, the answer
is
Problem 7.
Faces and of tetrahedron meet at an angle of . The area of
face is , the area of face is , and . Find the volume of the
tetrahedron.
Solution
Since the area , the perpendicular from to has length .
The perpendicular from to is . Therefore, the volume
is .
Problem 8.
For any sequence of real numbers , define to be the
sequence , whose term is . Suppose that all
of the terms of the sequence are , and that . Find .
Solution 1
Note that the s are reminiscent of differentiation; from the condition , we are led
to consider the differential equation This inspires us to guess a quadratic with leading
Thus, .
Solution 2
Let , and .
Note that in every sequence of
Then
Since
Solving, .
Solution 3
The sequence is the second finite difference sequence, and the first terms of
this sequence can be computed in terms of the original sequence as shown below.
Adding the above equations we find that
Solution 4
Since all terms of are 1, we know that looks like for some
. This means looks like . More
specifically, . Plugging in , we
have the following linear system: From this, we can
easily find that and . Solution by Zeroman
Problem 9.
Trapezoid has sides , , , and ,
with parallel to . A circle with center on is drawn tangent to and .
Given that , where and are relatively prime positive integers, find .
Solution 1
Let be the base of the trapezoid and consider angles and . Let and
let equal the height of the trapezoid. Let equal the radius of the circle.
Then
Let be the distance along from to where the perp from meets .
Solution 2
From above, and . Adding these equations yields .
Thus, , and .
We can use from Solution 1 to find that and .
This implies that so
Solution 3
Extend and to meet at a point . Since and are
parallel, . If is further extended to a point and is extended to a
point such that is tangent to circle , we discover that circle is the incircle of
triangle . Then line is the angle bisector of . By homothety, is the
intersection of the angle bisector of with . By the angle bisector theorem,
Let , then . .
Thus, .
Solution 4
both and have real and imaginary parts between and , inclusive. What is the integer
that is nearest the area of ?
Solution
We graph them:
We want the area outside the two circles but inside the square. Doing a little geometry, the
Problem 11.
Lines and both pass through the origin and make first-quadrant angles
of and radians, respectively, with the positive x-axis. For any line , the
transformation produces another line as follows: is reflected in , and the resulting line
is reflected in . Let and . Given that is the
Solution
Let be a line that makes an angle of with the positive -axis. Let be the reflection of in
, and let be the reflection of in .
The angle between and is , so the angle between and must also be .
Similarly, since the angle between and is , the angle between and the
positive -axis is .
Problem 12.
In a game of Chomp, two players alternately take bites from a 5-by-7 grid of unit squares. To
take a bite, a player chooses one of the remainingsquares, then removes ("eats") all squares
in the quadrant defined by the left edge (extended upward) and the lower edge (extended
rightward) of the chosen square. For example, the bite determined by the shaded square in
the diagram would remove the shaded square and the four squares marked by (The
squares with two or more dotted edges have been removed form the original board in previous
moves.)
The object of the game is to make one's opponent take the last bite. The diagram shows one
of the many subsets of the set of 35 unit squares that can occur during the game of Chomp.
How many different subsets are there in all? Include the full board and empty board in your
count.
Solution
By drawing possible examples of the subset, one can easily see that making one subset is the
same as dividing the game board into two parts.
One can also see that it is the same as finding the shortest route from the upper left hand
corner to the lower right hand corner; Such a route would require 5 lengths that go down, and
7 that go across, with the shape on the right "carved" out by the path a possible subset.
Problem 13.
Triangle has and . What's the largest area that this
triangle can have?
Solution
Solution 1
First, consider the triangle in a coordinate system with vertices at , , and .
Simplifying gives .
To maximize , we want to maximize . So if we can write: , then is
the maximum value of (this follows directly from the trivial inequality, because
if then plugging in for gives us ).
when .
Solution 3
Let be the endpoints of the side with length . Let be the Apollonian Circle of with
ratio ; let this intersect at and , where is inside and is outside. Then
because describes a harmonic
vertex to be the highest point of , which just makes the altitude have length . Thus,
Problem 14.
In triangle , , , and are on the sides , , and , respectively. Given
, find .
Solution
Using mass points, let the weights of , , and be , , and respectively.
Then, the weights of , , and are , , and respectively.
Thus, , , and .
Therefore:
Problem 15.
Define a positive integer to be a factorial tail if there is some positive integer such that the
decimal representation of ends with exactly zeroes. How many positive integers less
than are not factorial tails?
Solution
The number of zeros at the end
of is .
Note that if is a multiple of
, .
AIME 1994
AIME 1995
AIME 1996
AIME 1997
AIME 1998
AIME 1999
AIME 2000
AIME 2001
AIME 2002
AIME 2003
AIME 2004
AIME 2005
AIME 2006
AIME 2007
AIME 1986